add the missing sequence ​

Answers

Answer 1

Answer:

a) 2, 6, 10, 14, 18, 22

b)60, 59, 57, 54, 50, 45

c)240, 120, 60, 30, 15, 7 1/2

Step-by-step explanation:

The answer was already there.


Related Questions

2. Write the equation of the line in point-slope form.
(-1,3) and (2,9)

Answers

Answer:

y - 9 = 2 (x - 2)

Step-by-step explanation:

y2 - y1 / x2 - x1      9 - 3 / 2 - (-1)      6/3    = 2

y - 9 = 2 (x - 2)

18. The maintenance department ordered $3,450 worth of supplies from a valve and fitting supplier. The
supplier will allow a 15% discount because of the large order. How much will the maintenance department
have to pay for the supplies?
A. $2,932.50
B. $3,398.25
C. $3,406.45
D. $2,954.50

Answers

Answer:

A) [tex]\$\ 2932.5[/tex]

Step-by-step explanation:

One is given that a certain amount of money was allotted to be spent on supplies. However, there was a discount applied to the purchase. One is asked to find the amount of money actually spent on the supplies.

$3450 was the initial price that was to be spent on supplies, however, a (15%) discount was applied to this price. Subtract (15) from (100) to find the percent value that was actually spent on supplies.

[tex]100-15=85[/tex]

(85%) of the allotted money was actually spent on supplies. Now one has to find out the numerical value of the amount spent. Divide (85) by (100) and then multiply it by the amount of money allotted to the purchase, to fin the amount actually spent on the purchase.

[tex]3450*(\frac{85}{100})\\\\=3450*0.85\\\\=2932.5[/tex]

What is the length of the line?

Answers

The length of the line is B

Simplify Square root (150n^2)

Answers

Answer:

12

Step-by-step explanation:

Select the expression that has a value of 13.

9 + 3 x (2 ÷ 3) + 6
(9 + 3) x 2 ÷ 3 + 6
9 − (3 x 2) ÷ 3 + 6
(9 + 3 x 2) ÷ 3 + 6

Answers

Answer:

9 − (3 x 2) ÷ 3 + 6 is the answer

Please help me to find this answer

Answers

Step-by-step explanation:

angle of a triangle is 180, therefore to get the remaining one, subtract the sum of the two knows from 180, also for the second one; angle on a straight line is as well 180, since you have fine the interior one, subtract it from 180 to get the second answer

Answer:

so angles in a triangle add up to 180,

32+50+m<MQP=180

82+m<MQP=180

m<MQP=180-82

=98°

and angles on a straight line add up to 180 therefore

m<MQR=180-m<MQP

=180-98

=82

I hope this helps and if you don't understand feel free to ask

Define mean and median for a set of data

Answers

Answer:

The mean is the average number of a data set gotten by adding all the numbers then dividing by two.

the median is the middle number in a sorted set of data,either sorted in ascending order or descending order.

I hope this helps

To calculate mean:

• add up all the numbers,

• then divide by how many numbers there are.

Example:

Data: 5, 6, 10, 1

Add these all together, which is 22, then divide by 4 (because that is how many numbers there are), so:

the mean is 5.5

To calculate median:

Arrange your numbers in order.

Cross off numbers until you get to the middle.

If there is an even number of data, then find the two in the middle and find the middle of that.

Example:

Data: 5, 6, 10, 1

Arrange: 1, 5, 6, 10

So, the middle 2 are 5 and 6. In between these would be 5.5, so that is the median.

If the data was: 1, 5, 6, 8, 10,

then the median would be 6.

The volume of a rectangular prism (shown below) is 48x^3+56x^2+16x Answer the following questions:
(1) What are the dimensions of the prism?
(2) If x = 2, use the polynomial 48x^3+56x^2+16x to find the volume of the prism.
(3) If x = 2, use the factors found in part a to calculate each dimension.
(4) Using the dimensions found in part c, calculate the volume. Show all work.

Answers

Answer:

(a)

[tex]Length = 8x\\Width = 3x + 2\\Height = 2x + 1[/tex]

(b)

[tex]P(2) = 640[/tex]

(c)

[tex]Length= 16[/tex]

[tex]Width = 8[/tex]

[tex]Height =5[/tex]

(d)

[tex]Volume = 640[/tex]

Step-by-step explanation:

Given

[tex]P(x) = 48x^3 + 56x^2 + 16x[/tex]

Solving (a): The prism dimension

We have:

[tex]P(x) = 48x^3 + 56x^2 + 16x[/tex]

Factor out 8x

[tex]P(x) = 8x(6x^2 + 7x + 2)[/tex]

Expand 7x

[tex]P(x) = 8x(6x^2 + 4x + 3x + 2)[/tex]

Factorize

[tex]P(x) = 8x(2x(3x + 2) +1( 3x + 2))[/tex]

Factor out 3x + 2

[tex]P(x) = 8x(3x + 2)(2x + 1)[/tex]

So, the dimensions are:

[tex]Length = 8x\\Width = 3x + 2\\Height = 2x + 1[/tex]

Solving (b): The volume when [tex]x = 2[/tex]

We have:

[tex]P(x) = 48x^3 + 56x^2 + 16x[/tex]

[tex]P(2) = 48 * 2^3 + 56 * 2^2 + 16 * 2[/tex]

[tex]P(2) = 640[/tex]

Solving (c): The dimensions when [tex]x = 2[/tex]

We have:

[tex]Length = 8x\\Width = 3x + 2\\Height = 2x + 1[/tex]

Substitute 2 for x

[tex]Length=8*2[/tex]

[tex]Length= 16[/tex]

[tex]Width = 3*2+2[/tex]

[tex]Width = 8[/tex]

[tex]Height = 2*2 + 1[/tex]

[tex]Height =5[/tex]

So, we have:

[tex]Length= 16[/tex]

[tex]Width = 8[/tex]

[tex]Height =5[/tex]

Solving (d), the volume in (c)

We have:

[tex]Volume = Length * Width * Height[/tex]

[tex]Volume = 16 * 8 * 5[/tex]

[tex]Volume = 640[/tex]

PLS HELP
Find an equation of the line with a y-intercept of -3 and an x-intercept of -4.5

Answers

Answer:

y = - [tex]\frac{2}{3}[/tex] x - 3

Step-by-step explanation:

The equation of a line in slope- intercept form is

y = mx + c ( m is the slope and c the y- intercept )

Calculate m using the slope formula

m = [tex]\frac{y_{2}-y_{1} }{x_{2}-x_{1} }[/tex]

with (x₁, y₁ ) = (0, - 3) and (x₂, y₂ ) = (- 4.5, 0 ) ← coordinates of intercepts

m = [tex]\frac{0-(-3)}{-4.5-0}[/tex] = [tex]\frac{0+3}{-4.5-0}[/tex] = [tex]\frac{3}{-4.5}[/tex] = - [tex]\frac{2}{3}[/tex]

The line crosses the y- axis at (0, - 3 ) ⇒ c = - 3

y = - [tex]\frac{2}{3}[/tex] x - 3 ← equation of line

A random number generator is used to create a list of 300 single digit numbers

Answers

What are we trying to find?

Hi! I'd appreciate if you could help me on this question.

Liam is buying bottles of soda in packages that contain 8 bottles each. If the total number of sodas Liam bough t was between 45 and 50, how many did he buy? Explain your answer.

Answers

Answer:

48

Step-by-step explanation:

We need to find the multiples of 8

8,16,24,32,40,48

48 is between 45 and 50 so he must have bought 48

Answer:

6 bottles

Step-by-step explanation:

For this question we need to know the multiple of 8 which are:

8 x 1 = 8

8 x 2 = 16

8 x 3 = 24

8 x 4 = 32

8 x 5 = 40

8 x 6 = 48

8 x 7 = 56

There is only one multiple, which is greater than 45 but less than 50, which is 8x6 l.

This means he bought 6 bottles.

Answered by g a u t h m a t h

Prove that: sec⁴B - sec²B = tan⁴B + tan²B.​

Answers

Step-by-step explanation:

sec⁴B - sec²B = sec²B(sec²B - 1)

= (1 + tan²B)(tan²B)

= tan⁴B + tan²B

= Right-hand side (Proven)

Find the value of x.
A. 57
B. 72
C. 90
D. 124

Answers

Answer:

90

Step-by-step explanation:

Angle Formed by Two Chords= 1/2(SUM of Intercepted Arcs)

105 = 1/2 (120+x)

210 = 120+x

Subtract 120 from each side

210-120 = x

90 =x

The value of  Intercepted Arcs x will be 90. so option C is correct.

What is the relation between line perpendicular to chord from the center of circle?

If the considered circle has center O and chord AB, then if there is perpendicular from O to AB at point C, then that point C is bisecting(dividing in two equal parts) the line segment AB.

Or

|AC| = |CB|

Angle Formed by Two Chords= 1/2 (Sum of Intercepted Arcs)

105 = 1/2 (120+x)

210 = 120+x

Subtract 120 from each side;

210-120 = x

90 =x

Hence, the value of  Intercepted Arcs x will be 90. so option C is correct.

Learn more about chord of a circle here:

https://brainly.com/question/27455535

#SPJ5

simplify 27-{ 9+(12-5)÷4} with solution​

Answers

Answer:

16.25

Step-by-step explanation:

first do 12 -5 = 7. then 7/4 = 1.75 then 9+1.75 = 10.75 and finally 27-10.75= 16.25

This is how you do it

Help me please and thank you

Answers

Step-by-step explanation:

jlejej

are u using chrome os

Log 1 with whatever base (except 0) is 0 for any number (except 0) raised to 0th power is 1. And 0 times anything is 0. Answer should be B.

Solve for y. 14y-6(y-3)=22

Answers

Answer:

y=0.5

Step-by-step explanation:

14y-6(y-3)=22

14y-6y+18=22

8y+18=22

8y=4

y=0.5

Then we check our work...

14(0.5)-6((0.5)-3)=22

7-6(-2.5)=22

7+15=22

7+15 does equal 22, so this solution is correct.

1. In the past, Sam cashed his paycheck each month at Ready Cash, a check cashing service that
charges a 5% fee. He recently opened a checking account at Bank of America so he can now
deposit and/or cash his paycheck without a fee. If Sam is making $28,500 per year, how much will
he save by not going to Ready Cash anymore?

Answers

Step-by-step explanation:

28000 ÷ 100

=280

280 × 5

=1400

I need the answer explained

Answers

Answer:

1.33

Step-by-step explanation:

62 can only be subtracted from 82 once. So 82.46-62 would be 20.46. Since you can't subtract anymore you put a decimal point. 62x3=186 and 20.46-186=1.86 and you can subtract 186-186=0.

Please help!
The quantities x and y are proportional.
x: 4 5 10
y: 10 12.5 25
Find the constant of proportionality (r) in the equation y=rx.

Answers

9514 1404 393

Answer:

  r = 2.5

Step-by-step explanation:

The constant of proportionality can be found by solving the equation for r:

  r = y/x

Then any corresponding values of x and y can be used to find r:

  r = 25/10 = 2.5

The constant of proportionality is 2.5.

help! due august 12th

Answers

The output of the first one is 21 and you need to find x to the power of 2 subtract 4. So to find where we started add 4 to 21. This gets you to 25 and the square root of 25 is 5. So for the first one X=5

Work showed:

Y=x^2-4
21=x^2-4
Add 4 to both sides
25=x^2
Find the square root of 25 to get X as 5

For the second output (y), plug 21 for x

Y=4x+6
Y=4(21)+6
Y=84+6
Y=90

First part: X=5
Second part: Y=90

Please Answer This!!! I NEEEDDD TOOO KNOWWWWW ANSWER!!!

Answers

Answer:

77.5

Step-by-step explanation:

Its rising at a constant rate between +10-15 each hour, so we if we were to add 25 or so to the 50, it would be close to 77.5, so I would assume the answer was B

Find an equation of a plane containing the line r=⟨0,4,4⟩+t⟨−3,−2,1⟩ which is parallel to the plane 1x−1y+1z=−5 in which the coefficient of x is 1.
..?.. = 0.

Answers

The plane you want is parallel to another plane, x - y + z = -5, so they share a normal vector. In this case, it's ⟨1, -1, 1⟩.

The plane must also pass through the point (0, 4, 4) since it contains r(t). Then the equation of the plane is

x, y - 4, z - 4⟩ • ⟨1, -1, 1⟩ = 0

x - (y - 4) + (z - 4) = 0

x - y + z = 0

Surface Area of cones
Instructions: Find the surface area of each figure. Round your answers to the nearest tenth, if necessary.

Answers

9514 1404 393

Answer:

  64.1 ft²

Step-by-step explanation:

The area of the cone is given by ...

  A = πr(r +h) . . . . for radius r and slant height h

  A = π(2 ft)(2 ft +8.2 ft) ≈ 64.1 ft²

I need help on this problem​

Answers

9514 1404 393

Answer:

  see attached

Step-by-step explanation:

(a) The graph is scaled by a factor of 2, and shifted up 1 unit. The scaling moves each point away from the x-axis by a factor of 2. The points on the x-axis stay there. The translation moves that scaled figure up 1 unit.

__

(b) The graph is reflected across the x-axis and shifted right 4 units. The point on the x-axis stays on the x-axis.

Write the equation of the line that passes through the points (0, 4) and (- 4, - 5) . Put your answer in fully reduced slope intercept form , unless it is a vertical or horizontal line

Answers

Answer:

y=9/4x+4

Step-by-step explanation:

Start by finding the slope

m=(-5-4)/(-4-0)

m=-9/-4 = 9/4

next plug the slope and the point (-4,-5) into point slope formula

y-y1=m(x-x1)

y1=-5

x1= -4

m=9/4

y- -5 = 9/4(x - -4)

y+5=9/4(x+4)

Distribute 9/4 first

y+5=9/4x + 9

subtract 5 on both sides

y=9/4x+4

please explain it step by step​

Answers

6 is in the thousandth (1000) place

Mrs. Gomez has two kinds of flowers in her garden. The ratio of lilies to daisies in the garden is 5:2

If there are 20 lilies, what is the total number of flowers in her garden?

Answers

Answer:

28

Step-by-step explanation:

5 : 2

since this is a simplified ratio, they have a common factor. let's say it is 'x'

so now :

5x : 2x

we know that 5x is lilies, and we also know that she has 20 lilies, so:

5x = 20

x = 4

the daisies would be 2x so 2*4 = 8

total flowers is 20 + 8

28

If a and b are positive numbers, find the maximum value of f(x) = x^a(2 − x)^b on the interval 0 ≤ x ≤ 2.

Answers

Answer:

The maximum value of f(x) occurs at:

[tex]\displaystyle x = \frac{2a}{a+b}[/tex]

And is given by:

[tex]\displaystyle f_{\text{max}}(x) = \left(\frac{2a}{a+b}\right)^a\left(\frac{2b}{a+b}\right)^b[/tex]

Step-by-step explanation:

Answer:

Step-by-step explanation:

We are given the function:

[tex]\displaystyle f(x) = x^a (2-x)^b \text{ where } a, b >0[/tex]

And we want to find the maximum value of f(x) on the interval [0, 2].

First, let's evaluate the endpoints of the interval:

[tex]\displaystyle f(0) = (0)^a(2-(0))^b = 0[/tex]

And:

[tex]\displaystyle f(2) = (2)^a(2-(2))^b = 0[/tex]

Recall that extrema occurs at a function's critical points. The critical points of a function at the points where its derivative is either zero or undefined. Thus, find the derivative of the function:

[tex]\displaystyle f'(x) = \frac{d}{dx} \left[ x^a\left(2-x\right)^b\right][/tex]

By the Product Rule:

[tex]\displaystyle \begin{aligned} f'(x) &= \frac{d}{dx}\left[x^a\right] (2-x)^b + x^a\frac{d}{dx}\left[(2-x)^b\right]\\ \\ &=\left(ax^{a-1}\right)\left(2-x\right)^b + x^a\left(b(2-x)^{b-1}\cdot -1\right) \\ \\ &= x^a\left(2-x\right)^b \left[\frac{a}{x} - \frac{b}{2-x}\right] \end{aligned}[/tex]

Set the derivative equal to zero and solve for x:

[tex]\displaystyle 0= x^a\left(2-x\right)^b \left[\frac{a}{x} - \frac{b}{2-x}\right][/tex]

By the Zero Product Property:

[tex]\displaystyle x^a (2-x)^b = 0\text{ or } \frac{a}{x} - \frac{b}{2-x} = 0[/tex]

The solutions to the first equation are x = 0 and x = 2.

First, for the second equation, note that it is undefined when x = 0 and x = 2.

To solve for x, we can multiply both sides by the denominators.

[tex]\displaystyle\left( \frac{a}{x} - \frac{b}{2-x} \right)\left((x(2-x)\right) = 0(x(2-x))[/tex]

Simplify:

[tex]\displaystyle a(2-x) - b(x) = 0[/tex]

And solve for x:

[tex]\displaystyle \begin{aligned} 2a-ax-bx &= 0 \\ 2a &= ax+bx \\ 2a&= x(a+b) \\ \frac{2a}{a+b} &= x \end{aligned}[/tex]

So, our critical points are:

[tex]\displaystyle x = 0 , 2 , \text{ and } \frac{2a}{a+b}[/tex]

We already know that f(0) = f(2) = 0.

For the third point, we can see that:

[tex]\displaystyle f\left(\frac{2a}{a+b}\right) = \left(\frac{2a}{a+b}\right)^a\left(2- \frac{2a}{a+b}\right)^b[/tex]

This can be simplified to:

[tex]\displaystyle f\left(\frac{2a}{a+b}\right) = \left(\frac{2a}{a+b}\right)^a\left(\frac{2b}{a+b}\right)^b[/tex]

Since a and b > 0, both factors must be positive. Thus, f(2a / (a + b)) > 0. So, this must be the maximum value.

To confirm that this is indeed a maximum, we can select values to test. Let a = 2 and b = 3. Then:

[tex]\displaystyle f'(x) = x^2(2-x)^3\left(\frac{2}{x} - \frac{3}{2-x}\right)[/tex]

The critical point will be at:

[tex]\displaystyle x= \frac{2(2)}{(2)+(3)} = \frac{4}{5}=0.8[/tex]

Testing x = 0.5 and x = 1 yields that:

[tex]\displaystyle f'(0.5) >0\text{ and } f'(1) <0[/tex]

Since the derivative is positive and then negative, we can conclude that the point is indeed a maximum.

Therefore, the maximum value of f(x) occurs at:

[tex]\displaystyle x = \frac{2a}{a+b}[/tex]

And is given by:

[tex]\displaystyle f_{\text{max}}(x) = \left(\frac{2a}{a+b}\right)^a\left(\frac{2b}{a+b}\right)^b[/tex]

what is an example of a quintic bionomial?

Answers

3x^5+2

quantic means with the degree of 5 and binomial means two terms.

Scientists have steadily increased the amount of grain that farms can produce each year. The yield for farms in France is given by y=−2.73x2+11000x−11000000 where x is the year and y is the grain yield in kilograms per hectare (kg/ha).

What does the y-intercept of this function represent?

Answers

9514 1404 393

Answer:

  the yield in year 0

Step-by-step explanation:

The y-value is the yield for farms in France in year x. The y-value when x=0 is the yield for farms in France in year 0.

_____

Additional comment

The reasonable domain for this function is approximately 1843 ≤ x ≤ 2186. The function is effectively undefined for values of x outside this domain, so the y-intercept is meaningless by itself.

Other Questions
report on "road construction and its impact to public health in Nepal, Pokhara(please include the following areas : introduction ,objectives of the study , methodologies, findings, conclusions and recommendations ) Add.(-2+6x3 3x_) + (4x3 5 + x)Express the answer in standard form.Enter your answer in the box. WORTH 100 POINTS! The function h(x) is quadratic and h(3) = h(-10) = 0. Which could represent h(x)?1) h(x) = x2 - 13x - 302) h(x) = x2 - 7x - 303) h(x) = 2x2 + 26x - 604) h(x) = 2x2 + 14x - 60 Why is it difficult to bond three phosphate groups to Adenosine?They have the same charge and repel each other.They form a strong attraction to each other.They are like opposite poles of a magnet.The energy is evenly dispersed across all three bonds. Advantages of equity financing over debt financing include that: Multiple Choice equity financing does not require repayment. dividends are mandatory. stockholders' control will increase. dividends are tax deductible. In Washburn's factory, what is the break-even point for the new line of guitars if the retail price is (a) $349, (b) $389, and (c) $309? Also, (d) if Washburn achieves the sales target of 2,000 units at the $349 retail price, what will its profit be? A.) V (-3,-5), K (-1,-2), B (3,-1), Z(2,-5) B.) V(-4, 1), K(-2, 4), B(2,5) Z (1, 1)C.) V (-3,-4), K(-1,-1) B (3,0), Z(2,-4) D.) V (-1,0), K (1, 3), B(5,4), Z(4,0) Write a simplified polynomial expression in standard form to represent the area of the rectangle below:(See photo)A. 2x^2 + 3x - 20B. 2x^2 + 13x - 1C. 2x^2 + 13x - 20D. 2x^2 + 3x - 1 A professional painter and his apprentice, in business as a partnership, were hired to paint a store. Midway through the job they ran out of paint, so the painter lent his truck to the apprentice to pick up more. On his way to pick up the paint, the apprentice stopped at a post office along the way to mail a personal letter. On pulling out of the post office parking lot, he negligently ran into a parked car, causing extensive damage.If the car owner brings a negligence action against the painter, will she prevail?A. No, because the apprentice is an independent contractor.B. No, because a bailor is not vicariously liable for the torts of his bailee.C. Yes, because the apprentice's stop at the post office was not a frolic.D. Yes, because the apprentice was using the painter's truck. Illustrate the 7th pattern of the sequence of square numbers. what is meant by national project All of the following are true about a cyclical pattern EXCEPT it is ______. a. often combined with long-term trend patterns and called trend-cycle patterns b. often due to multi-year business cycles c. usually easier to forecast than a seasonal pattern due to less variability d. an alternating sequence of data points above and below the trend line The top part of Mars, Inc.'s 2018 balance sheet is listed as follows (in millions of dollars). Current assets: Current liabilities: Cash and marketable securities $ 10 Accrued wages and taxes $ 20 Accounts receivable 40 Accounts payable 30 Inventory 160 Notes payable 40 Total $ 210 Total $ 90 What are Mars, Inc.'s current ratio, quick ratio, and cash ratio for 2018? If per unit variable cost of a product is Rs.8 and fixed cost is Rs 5000 and it is sold for Rs 15 per unit, profit in 1000 units is.......a.. rs 7000b. rs 2000c. rs 25000d. rs 0 A study conducted by the University of Illinois in the 1950s found that pilots with insufficient instrument flying ability lose control of their airplane in an average of only ________ once they lose outside visual references Lee el consejo que le dio Martn a su prima._[blank]_ la ropa del mes pasado, promocinala por medio de ofertas.Qu opcin completa la oracin correctamente?Si quiere venderSi quieres venderS, quieres queSi quiere que 12) Find the angles between 0o and 360o where sec = 3.8637 . Round to the nearest 10th of a degree:Please show all work A cricketer throws a ball sideways with an initial velocity of 30 m/s. She releases the ball from a height of 1.3m. Calculate how far the ball travels before hitting the ground. We are given a weighted coin (with one side heads, one side tails), and we want to estimate the unknown probability pp that it will land heads. We flip the coin 1000 times and it happens to land heads 406 times. Give answers in decimal form, rounded to four decimal places (or more). We estimate the chance this coin will land on heads to What can be done to help maintain cardiovascular health?